• Shuffle
    Toggle On
    Toggle Off
  • Alphabetize
    Toggle On
    Toggle Off
  • Front First
    Toggle On
    Toggle Off
  • Both Sides
    Toggle On
    Toggle Off
  • Read
    Toggle On
    Toggle Off
Reading...
Front

Card Range To Study

through

image

Play button

image

Play button

image

Progress

1/63

Click to flip

Use LEFT and RIGHT arrow keys to navigate between flashcards;

Use UP and DOWN arrow keys to flip the card;

H to show hint;

A reads text to speech;

63 Cards in this Set

  • Front
  • Back

The first essay question had to do with a contract that was formed as follows:


On October 3, 2005, no longer able to obtain widgets from his regular suppliers, Bob called Sell Corp., a manufacturer of widgets. Bob Corp. placed an order with Sell Corp. for 10,000 widgets at a price of $1 per widget, to be delivered on October 31, 2005, with payment due on delivery.


On October 4, 2005, Sell Corp. sent the following "Order Confirmation" to Bob Corp., signed by Sara, as President:


“Sell Corp. accepts Bob Corp.’s order for 10,000 widgets, at $1 each, delivery on October 31, 2005, payment due on delivery.


Interest: Bob Corp. shall pay interest of 1½ % per month on payments made more than 10 days after delivery.


Waiver of Warranties: 24 hours after delivery all warranties of merchantability are waived by Bob Corp.”


There had been no discussion between the parties of any provision for interest or the waiver of warranties. On October 6, 2005, Bob Corp. received Sell Corp.’s "Order Confirmation." Bob Corp. never notified Sell Corp. that it was agreeing to or rejecting the provisions for interest on late payment or waiver of warranties and never signed a copy of the "Order Confirmation."

The question was whether there was an enforceable contract. I answered that there was offer and acceptance and considerationa and talked about the fact that there was a "battle of the forms" here. I was wrong! I didn't read carefully. He didn't send a form. He called! The answer was about the Statute of Frauds. See the following card....

. The issue is whether a contract between merchants satisfies the Statute of Frauds when the merchant to be charged did not sign the contract.


Contracts for the sale of goods must comply with Article 2 of the Uniform Commercial Code (“UCC”). Pursuant to the UCC, if the goods cost $500 or more, then the contract must also satisfy the Statute of Frauds. In many cases, a writing setting forth the intent of the parties and signed by the party to be charged will satisfy the Statute of Frauds. However, there is an exception to the signature requirement for contracts between merchants. If an oral agreement between two merchants is followed by written confirmation of the agreement and signed by the sender of the confirmation, then the recipient will also be bound by the confirmation if he had reason to know of its contents and fails to object to it within 10 days.


Here Bob Corp. is in the business of buying and selling widgets and Sell Corp. is a manufacturer of widgets. Both parties are merchants. Bob Corp. placed an order for 10,000 widgets at $1 per widget but the order was not in writing. Sell Corp. responded to the order with a written “Order Confirmation” signed by Sara, president of Sell Corp. Bob received the Order Confirmation but never signed it. Because he failed to object to the contents of the Order Confirmation within 10 days, he is deemed to be bound by its terms. Therefore, the contract between Bob Corp. and Sell Corp. is enforceable.

2. (a) The issue is whether a term permitting interest on late payments added to an acceptance between merchants is enforceable. A valid contract requires an offer, acceptance, and consideration. As previously mentioned, Article 2 of the UCC governs transactions involving the sale of goods. At times, the UCC applies different rules to merchants—those who regularly deal in the kinds of goods involved in the transaction—than to non-merchants. Under the UCC, when at least one party to a transaction for the sale of goods is not a merchant, an acceptance that adds additional terms to the contract results in a valid and enforceable contract, but that contract will not include the additional terms.


The rule is different when both parties to the transaction are merchants: an acceptance with additional terms creates an enforceable contract and the additional terms will automatically become part of the contract unless: (i) they materially alter the contract, (ii) the offer limits acceptance to the terms as stated, or (iii) the offeror has objected to or objects to the additional terms within a reasonable time.

Under the facts, Bob Corp.’s order did not expressly limit acceptance to the terms of his order. Further, Bob Corp. never objected to the additional term. Thus, the only issue is whether the interest penalty on late payments is a material alteration to the terms of the contract. Interest on late payments does not alter the rights or obligations of either side, and is customary in many businesses. Further, the interest on late payments is avoidable by timely payment. Consequently, the inclusion of the term is not a material alteration, and thus, is enforceable.

2. (b) The issue is whether a waiver of the right to enforce the warranty of merchantability is enforceable. Implicit in every sale of goods contract in which the seller is a merchant is a warranty of merchantability that promises that the items sold will work in accordance with expectations. Waiving the right to a guarantee that items purchased will work properly is foregoing a significant right under the contract. For this reason, any written disclaimer is required to be conspicuous within the contract. Consequently, a court would conclude that adding such a waiver to a contract by way of a merchant’s acceptance is a material alteration to the contract that is unenforceable.

-

2. (c) Can Sell Corp. sue Bob personally for the debt Bob Corp. owes Sell Corp? In order to be successful, Sell Corp. will be have to pierce the corporate veil in order to sue Bob, the sole shareholder and sole director of Bob Corp., in his individual capacity. Under New York law, corporations are separate entities from the shareholders who own them, and any liability remains solely that of the corporation, and not the responsibility of individual shareholders. However, in New York, the limited liability protecting shareholders will be disregarded if it is necessary to prevent fraudulent, inequitable, or illegal conduct, and the plaintiff is successful in demonstrating the disregard of corporate formalities, the excessive domination by shareholders, that the business is being carried on for personal gain, or that the corporation is being used to hide illegal business.

Here, Bob Corp. was insolvent when it placed its order, and it appears that Bob created the corporation to avoid suit for nonpayment of suppliers. In addition, Bob disregarded corporate formalities by failing to keep or maintain corporate records and by commingling his personal and business accounts. Consequently, it is likely Sell Corp. will be successful in demonstrating that piercing the corporate veil and holding Bob individually responsible is necessary to prevent fraud.

When can a mistake allow a party to void a contract?

When one party is mistaken as to an essential element of the contract, the mistaken party can void the contract if she did not bear the risk of mistake and the non-mistaken party caused the mistake.

Under the UCC, what elements of a contract can be missing and what elements must be there?

parties, subject matter and quantity must be there, but price can be supplied by the court if parties can't agree

How do firm offers work? Does a condition in a firm offer make it just a regular offer?

Generally, unless consideration is given to keep an offer open, the offeror can withdraw an offer at any time prior to its acceptance by the offeror, even an offer that the offeror states is irrevocable. However, a firm offer made by the offeror in an authenticated writing is irrevocable for the stated time period, where such period is 90 days or less, provided that the offeror is a merchant. Even if there's a condition in the offer like "if it's in stock" - as long as the condition is met (the car was in stock, for example)

A wholesaler of bicycle chains sent a retailer the following fax on December 1: “Because of your continued loyalty as a customer, I am prepared to sell you up to 1,000 units of Bicycle Chain Model D at $7.50 per unit, a 25% discount off our original $10.00 price. This offer will remain open for 7 days.” The fax was not signed, but was on the wholesaler’s letterhead and had been initialed by the wholesaler’s head of sales. On December 4, the wholesaler’s head of sales called the retailer and informed the retailer that he had decided to revoke his December 1 offer. On December 5, the retailer placed an order for 1,000 bicycle chains, stating that he would pay the discounted price of $7.50 per unit.

What is the correct value of the order placed by the retailer?


A. $7,500, because the wholesaler’s revocation was not in writing.B. $7,500, because the wholesaler was bound to keep the offer open for 7 days.C. $10,000, because the offer was not signed by the wholesaler.D. $10,000, because the retailer did not provide consideration to hold the offer open.

Answer choice B is correct. Under the UCC’s firm offer rule, an offer to buy or sell goods is irrevocable if the offeror is a merchant, there are assurances that the offer is to remain open, and the assurance is contained in an authenticated record (such as a signature, initials, or other inscription) from the offeror. No consideration by the offeree is needed to keep the offer open. Here, all three conditions are satisfied, and the wholesaler’s offer was irrevocable until the offer period expired. The retailer’s order was within the 7-day window, and thus the order for 1,000 units is priced at $7.50 each, or $7,500 total. Answer choice A is incorrect because the wholesaler was bound by the UCC firm offer rule, and his offer was irrevocable regardless of the method of his attempted revocation. Answer choice C is incorrect because, although a firm offer must be contained in an authenticated record, the letterhead and initials of the head of sales were sufficient to authenticate the writing in this case. Answer choice D is incorrect because, under the UCC’s firm offer rule, no consideration was required to hold the offer open.

A manufacturer of plywood ordered an adhesive from a commercial seller. The seller shipped the adhesive with an invoice. Printed on the invoice was a notice that limited the manufacturer’s right to bring any action arising from the sale to one year after the accrual of the cause of action. The manufacturer paid the invoice without objection. The statutory period for bringing a cause of action based on sale of goods contracts is four years. The manufacturer filed a suit that arose from the contract two years after the cause of action accrued.

Can the seller successfully challenge the suit as time barred?


A. Yes, because the manufacturer failed to object to the invoice notice.B. Yes, because merchants are free to set whatever limitations period for bringing an action that arises from a sale of goods.C. No, because the one-year limitation term is not part of the contract.D. No, because the statutory limitations period cannot be shortened.

Answer choice A is correct. As between merchants, an additional term contained in the acceptance is automatically included in the contract unless the offer expressly limits acceptance to the terms of the offer, the offeror has already objected to the additional terms or does so within a reasonable time, or the term materially alters the original contract. A seller can accept a buyer’s offer by shipping the goods, which the commercial seller did in this case.

A carpenter is in a contract to build cabinets for 25k. He discovers there's more work to be done than originally discussed and asks for $5k more. The homeowner verbally agrees. Is this modification of the contract enforceable?

Yes. His increased work is consideration for the increased cost and she verbally agreed, which is ok under common law, which is the law applied here under the predominant source rule - this is mostly a services contract not a goods contract.

A widow offered to sell her deceased husband’s wedding ring to a friend at its fair market price. Although the widow did not objectively indicate otherwise, the friend knew that the widow would never actually be able to part with the ring. Regardless, the friend agreed to buy the ring.
Does the friend’s agreement constitute a valid acceptance of the widow’s offer?


A. Yes, because the price for the ring was reasonable.B. Yes, because, as judged objectively, the widow offered to sell the ring.C. No, because the widow lacked the subjective intent to sell the ring.D. No, because the friend was aware that the widow lacked the subjective intent to sell the ring.

Answer choice D is correct. Although in contract law, intent is generally determined by an objective standard and not by the subjective intent or belief of a party, when an offeree is aware of the other’s party’s subjective intent not to enter into a contract, the offeree’s agreement to enter into a contract does not constitute a valid acceptance. Answer choice A is incorrect. While the price to be paid for a good is an indication of the owner’s intent to sell it, the reasonableness of the sale price does not override the putative offeree’s knowledge that the owner lacks the subject intent to sell the good.

An independent truck driver often received assignments to transport cargo for a delivery company. The driver was one of the delivery company's most reliable contractors. When the driver was in need of a new truck, the delivery company contracted with a truck manufacturer to purchase the truck on an installment basis. The manufacturer retained a security interest in the truck until all payments were made. The contract between the delivery company and the manufacturer provided that the delivery company "shall not assign this contract without the prior written consent" of the manufacturer. Nonetheless, the delivery company assigned the contract, in writing, to the truck driver. The truck driver made all payments on the truck for two years until she was involved in a serious collision that destroyed the truck. The insurance proceeds on the destroyed truck were paid to the manufacturer. After the manufacturer used the proceeds to satisfy the contract balance, $20,000 remained. The truck driver delivered a copy of the assignment to the manufacturer, and demanded the remainder of the proceeds. The truck manufacturer stated that the contract was non-assignable, and that it would only pay the proceeds to the delivery company. The truck driver then filed suit to compel the truck manufacturer to pay her the proceeds.

Is she likely to succeed?


A. Yes, because the written assignment from the delivery company to the truck driver was a novation.B. Yes, because the contract was assigned to the truck driver.C. No, because the contract between the delivery company and the truck manufacturer was non-assignable.D. No, because the truck driver was merely an incidental beneficiary of the contract between the delivery company and the truck manufacturer.

Answer choice B is correct. Most contracts can be assigned. Even if the contract by its terms prohibits assignment, a party retains the power to assign, although an assignment operates as a breach of the contract. Consequently, the truck manufacturer has a claim against the delivery company, but cannot refuse to recognize the assignment to the truck driver. Consequently, it must disburse the remainder of the insurance proceeds to her. For that reason, answer choice C is incorrect. Answer choice A is incorrect because a novation is the substitution of a new contract for an old one; in this case, a novation could only have occurred with the consent of the truck manufacturer, which was not sought, much less given. Answer choice D is incorrect because the truck driver's status as either an intended or incidental beneficiary of the contract does not bear upon the validity of the assignment of the contract to her.

An electrician and a landlord executed a contract under which the electrician agreed to upgrade the electricity in the landlord’s ten-unit apartment building at a cost of $10,000. The electrician began work as scheduled and completed two of the ten identical units before quarrelling with the landlord. The electrician refused to complete the job. The landlord hired another electrician who completed the last eight units at $1,100 each in the same time frame as was contemplated in the landlord’s contract with the electrician.
If the first electrician sues the landlord for restitution damages, how much will a court likely award?


A. $9,200B. $2,000C. $1,200D. $0

Answer choice C is correct. Restitutionary damages restore to the plaintiff (here, the electrician) whatever benefit was conferred upon the defendant (here, the landlord) prior to the breach. If the plaintiff breached the contract, his damages are generally limited to the amount of wealth conferred upon the defendant, which would take into consideration any damages suffered by the defendant. In this case, the electrician conferred roughly $2,000 of benefit on the landlord (two-tenths of the value of the original contract, assuming the contract price reflected the value conferred), and the landlord incurred $800 in extra costs as a result of the electrician’s breach (an extra $100 per unit x eight units). Accordingly, the electrician is entitled to $1,200, or the amount of benefit the electrician conferred upon the landlord. Answer choice A is incorrect because the electrician was in breach and did not substantially perform his contractual obligations. Consequently, he is not entitled to recover the full contract price ($10,000) less damages incurred by the landlord because of the electrician’s breach ($100 x 8, or $800). Answer choice B is incorrect. Had the electrician sought expectation damages under the contract, he may have been entitled to $2,000 if it was determined that the contract was divisible, because he completed two out of the ten units. However, this amount fails to take into account the cost of completion. Answer choice D is incorrect because the electrician would be entitled to recover some amount of damages, namely the amount of wealth incurred upon the landlord.

A farmer contracted with a fruit distributor to sell the distributor 1,000 bushels of peaches at $10 per bushel, the going market price for peaches. After the parties executed the contract, a late frost in a neighboring state completely destroyed that state’s peach crop. The affected state normally produced over half the peaches grown in the country. In response, the market price of peaches spiked to over $50 per bushel. The farmer attempted to renegotiate with the distributor, who refused to modify the contract. Thereafter, the farmer refused to deliver the peaches as scheduled. The distributor sued the farmer, who raised the defense that to force him to sell his produce so far below market value would be unconscionable.
Does the farmer have a valid defense?


A. Yes, because no reasonable person would agree to sell peaches at one-fifth of market price.B. Yes, because the frost was unforeseeable.C. No, because the farmer and the distributor had fairly equal bargaining power.D. No, because the contract price was reasonable when the farmer agreed to it.

Answer choice D is correct. The UCC provides that a court may modify or refuse to enforce a contract or part of a contract on the ground that it is unconscionable, that is, when it is so unfair to one party that no reasonable person in the position of the parties would have agreed to it. The contract or part of the contract at issue must have been offensive at the time it was made. Here, the farmer contracted with the distributor to sell peaches at $10 per bushel, which was a fair market price at the time the parties entered the contract. The fact that intervening events caused prices to rise only means that the farmer did not receive as good a deal as he could have if he had waited; this, however, is the risk inherent in business dealings and does not make the transaction unconscionable. Answer choice A is incorrect because this addresses only half of the analysis; the unfairness must also exist at the time of contracting. Answer choice B is incorrect because foreseeability is not a relevant factor in an unconscionability inquiry. Answer choice C is incorrect because, while the defense of unconscionability is often used when there is a contract between parties with vastly unequal bargaining power, that is not the only way that a contract could be unconscionable. In other words, this contract still could have been unconscionable, even if the bargaining power of the two parties was relatively equal.

An art collector selling a painting at auction can withdraw the item (can't bear to part with it) ...

at any time before the gavel falls on the highest bid for that specific painting - not the end of the whole auction event

If a lumberyard enters into a contract with me to provide lumber for house I'm building as a contractor and delegates its duty under the contract to provide certain lumber to me and I've agreed to the delegation, if the third party (the delegatee) breaches, is the lumberyard on the hook?

yes - the delegation of duties doesn't get them out of the contractual obligations

A homeowner met with a contractor regarding remodeling the homeowner’s kitchen. At the conclusion of their meeting, the contractor told the homeowner that he would charge her $9,000-$10,000 for the work, but that he would get back to her with a definite price once he returned to his office. When he arrived at his office, the contractor had a voicemail from the homeowner, saying that she would pay him $9,000 for the work they discussed. The contractor promptly returned her phone call, and left her a voicemail saying that he would do the work for $9,500, which the homeowner received. The next day, the contractor, having a change of heart, tried unsuccessful to reach the homeowner by phone. He left her a voicemail that he would do the work for $9,000 after all, and that he would start the following day unless he heard otherwise from her. The next day, the contractor showed up at homeowner’s house, ready to begin.
What best describes the relationship between the parties?


A. A contract was formed when the contractor left the final voicemail.B. A contract was formed when the homeowner received the final voicemail.C. A contract was formed when the contractor showed up to begin work on the kitchen.D. There is no contract between the parties.

Answer choice D is correct. An offer is terminated by rejection. A modification of the terms of the offer acts as a rejection of the original offer and as a new counteroffer. In this case, the contractor’s first voicemail served as a rejection of the homeowner’s original offer of $9,000 and a counteroffer of $9,500. The original offer was terminated; the contractor could not later accept the homeowner’s offer. Accordingly, no contract was formed. Answer choices A, B, and C are incorrect because the contractor rejected the homeowner’s offer and thus there was no contract.


An honest dispute develops between a homeowner and an electrician over whether wiring and circuit breakers installed by the electrician satisfied contractual specifications. If the wiring and circuit breakers meet those specifications, the homeowner owes the electrician $10,000 under the terms of the contract. The homeowner offers to pay the electrician $8,000 in satisfaction of the homeowner's contractual obligations, if the electrician replaces the circuit breakers with a different brand. The electrician accepts the homeowner's offer. After the electrician replaces the circuit breakers, the homeowner refuses to pay the electrician. In a breach of contract action brought by the electrician, the fact-finder determines that the wiring and circuit breakers originally installed by the electrician did satisfy the contract specifications. The fact-finder also determines that the electrician and the homeowner entered into an accord for which the homeowner failed to prove the required satisfaction.

What is maximum amount that the electrician can seek in damages from the homeowner?


A. $18,000B. $10,000C. $8,000D. Nothing

Answer choice B is correct. Since the electrician and the homeowner entered into an accord for which the homeowner failed to provide the required satisfaction, the electrician may seeks damages under the accord of $8,000 or may seek damages under the original contract of $10,000. Since $8,000 is less than $10,000, answer choice C is incorrect. Answer choice A is incorrect because, even though the homeowner has breached both the original contract and the accord by failing to pay the electrician, the electrician must elect to receive damages under either the original contract or the accord. Answer choice D is incorrect since the homeowner has breached both the accord and the original contract.


A grocery store and a farmer entered into a valid contract for 1,200 bottles of Grade A maple syrup, to be delivered over the course of a year in twelve equal installments of 100 bottles, delivered on or before the last day of each month, beginning in January and ending in December. From January to March the farmer delivered three shipments of conforming Grade A maple syrup. In April, on the last day of the month, the farmer delivered to the store 97 bottles of Grade A maple syrup. The grocery store rejected the syrup, and informed the farmer that the contract between them was canceled.
Which of the following is a correct statement of the grocery store’s actions?


A. The store’s rejection of the 97 bottles of Grade A maple syrup and cancellation of the contract were proper.B. The store’s rejection of the 97 bottles of Grade A maple syrup was proper, but the store’s cancellation of the contract was improper.C. The store’s rejection of the 97 bottles of Grade A maple syrup was improper, but the store’s cancellation of the contract was proper.D. The store’s rejection of the 97 bottles of Grade A maple syrup and cancellation of the contract were improper.

D - UCC allows substantial performance for installment contracts, so can't reject the shipment and this slight deficiency does not "substantially impair the value of the entire contract" so the store can't cancel the contract.

A customer entered a hardware store to purchase paint. The associate who helped the customer had just been hired the day before, and was not trained with regard to paint selection. The customer explained that she was painting her house and had applied a water-based primer, and that she needed to select an appropriate top coat. The associate showed the customer an oil-based paint, assuring the customer that the paint was appropriate for the job. The associate then told the customer that he was selling the paint “as is,” and that he could not be responsible for any adverse reactions. The customer made the purchase and used the oil-based paint to paint her house. Upon drying, the paint immediately peeled away from the water-based primer, causing extensive damage to the exterior of the house.

In a breach of warranty action against the hardware store, will the customer prevail?


A. Yes, because the associate’s promises created an express warranty.B. Yes, because the warranty of fitness for a particular purpose cannot be disclaimed.C. No, because the associate’s statement that the paint was sold “as is” disclaimed any warranties.D. No, because the associate was not a merchant.

Answer choice A is correct. Any promise, affirmation, description, or sample that is part of the basis of the bargain is an express warranty, unless it is merely the seller’s opinion or commendation of the value of the goods. In this case, the associate’s assurances that the paint selected was appropriate for the job created an express warranty. That warranty was breached, and the customer would prevail. Answer choice B is incorrect because the implied warranty of fitness for a particular purpose can be disclaimed by use of language such as “as is,” which calls the buyer’s attention to the exclusion of warranties and makes plain that there is no implied warranty. However, such disclaimer must be in writing and conspicuous. Answer choice C is incorrect because disclaimer clauses that conflict with express warranties are ignored. Thus, any attempt to disclaim the warranty with the language “as is” would be ignored. Answer choice D is incorrect because express warranties may be made by a merchant or a non-merchant. Moreover, the associate in this case was a merchant.


An elderly woman and her friend went to a sewing shop frequented by quilters to buy a sewing machine. The salesperson showed the woman several models and allowed the woman to test the various units with small swatches of fabric. After using one machine, the woman turned to her friend and exclaimed, “This one sews like a dream; I can’t wait to start quilting on it.” The salesperson, who overheard the woman, knew that the sewing machine was appropriate only for lightweight fabrics and was not powerful enough for quilting, but said nothing. When the woman decided to purchase the machine, the salesperson informed her that all sewing machine sales were final. The salesperson encouraged the woman to read the machine’s user’s manual, but the woman replied, “Oh, I’ve given the machine a test run, and I’m happy with it. Besides, I haven’t brought my reading glasses and I can’t make out the tiny text in the user’s manual.” The woman purchased the sewing machine and was given a sales receipt that was stamped “FINAL SALE.” When she brought the machine home, the woman realized the machine was not appropriate for quilting, and took it back to the store. The salesperson referred to the store’s final sale policy and refused the return.

Does the woman have a valid claim against the sewing shop?


A. Yes, because the salesperson owed a fiduciary duty to the woman.B. Yes, because the salesperson failed to correct the woman’s mistaken belief that the sewing machine was appropriate for quilting.C. No, because the salesperson made clear that the sale was final.D. No, because the salesperson did not make any misleading statements regarding the machine.

Answer choice B is correct. A fraudulent misrepresentation gives the person defrauded the chance to avoid a contract with the person who made the fraudulent assertion. Nondisclosure of a known fact is tantamount to an assertion that the fact does not exist, if the party not disclosing the fact knows that disclosure would correct a mistake of the other party as to a basic assumption of the contract, and the failure to disclose would constitute lack of good faith and fair dealing. Here, the salesperson (1) made a false assertion about an existing fact (non-disclosure to the elderly woman of the fact that the sewing machine could not handle quilts); (2) which was fraudulent (i.e., knowingly and intentionally misleading) or at least material; and (3) the elderly woman actually and justifiably relied on the salesperson’s assertion in entering the contract. Accordingly, the woman would be able to avoid the store’s final sale policy.

Answer choice A is incorrect because the salesperson owed a duty not to misrepresent facts to the woman, but not a fiduciary duty, to the woman. Answer choice C is incorrect because, although the salesperson informed the woman that the sale was final, the salesperson’s bad-faith nondisclosure rendered the sale voidable at the woman’s election. Answer choice D is incorrect because nondisclosure of a known fact is treated as an assertive misrepresentation in this case.

The owner of a fur coat stored it with the furrier from whom she bought the coat during the warm months of the year. While the coat was at the furrier, a salesperson, mistakenly thinking that the coat was for sale, sold it to a customer. The customer was allowed to reduce the purchase price by the amount of an outstanding debt owed by owner of the furrier to the customer; the customer paid the remainder in cash. In the process of purchasing the coat, the customer was told by the salesperson about the furrier's storage service, but, like the salesperson, was unaware that the coat was not part of the store's merchandise. After the sale, the owner learned of the transaction between the furrier and the customer. Since the coat had significant sentimental value to the owner, the owner sought its return from the customer. When the customer refused, the owner filed an action to recover the coat from the customer.

Will the owner prevail?


A. Yes, because the furrier transferred only voidable title in the coat to the customer.B. Yes, because the customer did not give full value in acquiring the coat.C. No, because the customer was a good faith purchaser of the coat that had been entrusted to the furrier.D. No, because the owner is entitled to damages from the furrier.

Answer choice C is correct. A good faith purchaser of goods in the ordinary course of business from a merchant takes good title to the goods if the goods have been entrusted by the owner to the merchant and the merchant deals in the same kind of goods. Here, the customer bought the fur coat at a furrier where it had been left by the owner. The customer was unaware that the coat belonged to the owner. Answer choice A is incorrect because although the furrier did not have title to the goods, the furrier could nevertheless transfer good title. Answer choice B is incorrect because the purchaser did give value: satisfaction of a pre-existing debt constitutes value. Answer choice D is incorrect because the owner's right to seek damages from the furrier does not, in itself, preclude the owner from seeking to recover the coat from the customer. The entrustment of the coat to a furrier and the subsequent sale of that coat to a good faith purchaser, however, does.

A math major at a local college agreed to tutor a high school student who was having trouble in his math class. The tutor promised to meet with the student for five hours a week for the remaining 2 months of the term, and the student’s mother agreed to pay the tutor $20 per hour. In addition, the parties agreed that, if the student received a B or better in his math class, the mother would pay the tutor a $500 bonus. The parties did not reduce their agreement to writing. The mother paid the tutor weekly for the tutoring. At the end of the term, the student received a B+ in his math class. The mother, not having the funds to pay the tutor’s bonus, contacted the tutor and offered her a bike worth $300 in lieu of the bonus payment. The tutor accepted the bike.
Is the tutor entitled to recover the remaining $200?


A. No, because the tutor accepted the bike instead of the bonus.B. No, because the tutor agreed to accept a lesser amount.C. Yes, because the modification was not in writing.D. Yes, because there was no dispute as to the amount owed.

Answer choice A is correct. Under an accord agreement, one party to a contract agrees to accept a different performance from the other party than what was promised in the existing contract. Although the original debt is not generally discharged immediately upon entering into an accord, once the creditor accepts the lesser amount offered by the accord, the original contract is discharged. Note, however, that agreeing to accept a different type of performance does discharge the original contract. Here, by agreeing to accept the bike as substitute performance (and by actually accepting the bike), she forfeited her right to recover the remaining $200. Answer choice B is incorrect because simply agreeing to an accord does not automatically discharge the original obligation. Until satisfaction occurs, the creditor can still recover under the original contract, and the original debt is not discharged unless there is a dispute as to the validity of the debt or when the payment is of a different type than called for under the original agreement. Here, even though the original agreement was discharged (both because satisfaction occurred and because payment was of a different form than originally contemplated), the tutor’s agreement to a lesser amount is not what discharged that original agreement. Answer choice C is incorrect because there is no requirement that contract modifications or accords be in writing, unless they fall within the Statute of Frauds. Answer choice D is incorrect. Whether there is a dispute as to the amount owed is relevant to whether an original debt is discharged; here, the fact that there was no dispute as to the amount owed would actually support the mother’s argument that she fully performed on the contract. However, the reason that the tutor is not entitled to recover the remaining $200 is that she already accepted the bike in lieu of the $500 bonus payment.

A contract entered into under duress is ___________, unless the duress is a threat of physical violence, in which case the contract is _________.

voidable


void

A homeowner entered into oral contracts with both a painter and a landscaper to perform services at his home. The landscaper was the first to begin the services, and shortly after he began to work, he realized that the type of soil in the homeowner’s yard would cause the projected cost of the work to increase dramatically. After the homeowner realized how high the cost of the landscaping services were going to be, he called the painter to tell her that he could not go through with their contract at that time. The painter stated that she had already purchased paint and brushes for the job, as well as painted glass to create a small, artistic mosaic on a back corner of the house as a sort of signature expression she planned to begin using. She had also paid for a temporary city permit to park her utility van on the residential street where the homeowner lived.
Which of the following would not be a possible liability for the homeowner?


A. The contract price minus the market cost of performanceB. Cost of the paint and brushesC. Cost of the glassD. Cost of the permit

Answer choice C is correct. Non-breaching parties may often choose between several different types of damages, including reliance damages (those the plaintiff reasonably incurred in reliance of the contract) and expectation damages (which put the injured party in the position that she would have been in if not for the breach). To calculate expectation damages, the market value of performance should be subtracted from the contract price. Expectation damages must be foreseeable, so the glass tiles for the mosaic would not fall under the umbrella of expectation damages, as there was no way for the homeowner to know she was using them. Incidental damages may also be recovered for commercially reasonable expenses incurred as a result of the other party’s breach, and the cost of the permit would fall under this category. Note, however, that parties cannot recover both reliance and expectation damages, so the painter would not be able to recover both the contract price and the price of supplies. Answer choice A is incorrect because it reflects expectation damages, which is one of the possible remedies for the non-breaching party. Answer choice B is incorrect because the cost of supplies could constitute reliance damages. Answer choice D is incorrect because the painter incurred the cost in anticipation of completing work at the homeowner’s residence, and she would be entitled to reimbursement for that expense incurred in reliance on the contract.

The owner of undeveloped land entered into a fully-integrated written contract to sell the land to a buyer for $500,000. The contract contained a description that accurately set out the boundaries of the land, but did not make reference to the specific acreage. Both the owner and the buyer mistakenly thought that the property encompassed 500 acres, on the basis of a negligent miscalculation made by a surveyor employed by the owner and contained in the surveyor’s report. In fact the property encompassed 550 acres. The owner had sought, but the buyer had rejected, the inclusion of a provision in the contract that, in the event of a material error in the surveyor’s report, the adversely affected party could cancel the contract.

Can the owner cancel the contract?


A. Yes, because both the owner and seller were mistaken as to the land’s acreage.B. Yes, because the parol evidence rule does not prevent the introduction of evidence of mistake.C. No, because the mistake was made by a surveyor employed by the owner.D. No, because the owner assumed the risk of the mistake.

Answer choice D is correct. The owner and the seller were mistaken as to the acreage being transferred, and this mistake is arguably material, since it constitutes 10 percent of the amount the parties thought was being transferred. However, the defense of mistake is not available where the party seeking to assert the mistake has assumed the risk of the mistake. The owner’s failed attempt to have a provision included in the contract that would have given the owner the right to cancel the contract because of the surveyor’s error indicates that the owner was aware of his limited knowledge as to the accuracy of the surveyor’s report and that he entered into the contract despite such awareness. In the face of such conscious ignorance, the owner bears the risk of the mistake. Answer choice A is incorrect because, as noted with respect to answer choice D, even though the owner and the seller were mistaken as to the acreage being transferred, the defense of mistake is not available when the party seeking to assert the mistake has assumed the risk of the mistake. Answer choice B is incorrect because, although the parol evidence rule does not prevent the introduction of evidence of mistake, the owner cannot successfully assert a mistake defense. Answer choice C is incorrect because a party to a contract may assert mistake as a defense, even where the party or an agent of the party is the source of mistake, unless the mistake constitutes a failure to act in good faith and in accordance with the reasonable standards of fair dealing.

A plastics manufacturer saw an advertisement for a plastic extruding machine. The manufacturer contacted the seller and made arrangements to inspect the machine at the seller’s place of business. The manufacturer walked around the machine once and stated: “Yes, this looks like what I need.” When the manufacturer asked the price, the seller stated a price that was less than half the amount a similar, functioning, used machine commanded on the market. The manufacturer was surprised at the low price, but did not inquire as to the reason. The seller encouraged the manufacturer to perform a closer inspection before finalizing the purchase and offered to open the motor housing so that the motor could be examined, but the manufacturer declined. The parties completed the sale. The manufacturer transported the extruding machine to his factory. When it arrived, he first learned that the motor was burned out and required complete replacement, as was readily apparent upon visual inspection of it. Replacement of the motor would cost roughly the amount the manufacturer had paid for the machine. The manufacturer contacted the seller to return the machine, but the seller refused. The manufacturer filed suit against the seller.


Will the manufacturer prevail?


A. Yes, because the seller violated the implied warranty of merchantability by selling a machine with a burned-out motor.B. Yes, because the manufacturer’s unilateral mistake regarding the condition of the machine was caused by the seller.C. No, because the seller made no claims regarding the operability of the machine.D. No, because the manufacturer waived any implied warranties by failing to inspect the machine.

Answer choice D is correct. Under UCC Article 2, a warranty of merchantability is implied whenever the seller of goods is a merchant. To be merchantable, goods must be fit for their ordinary purpose and pass without objection in the trade. A breach of this warranty must have been present at the time of the sale. However, if the buyer, before entering into the contract, has examined the goods as fully as the buyer desires, or has refused to examine the goods, there is no implied warranty with respect to defects that an examination would have revealed to the buyer. Here, the manufacturer declined to closely inspect the machine, even after learning of the unusually low sales price. Had he done so, he would have discovered the damaged motor when he opened the motor housing, which the seller had offered to do. Because the manufacturer refused to examine the goods, he waived any implied warranty of merchantability that would have otherwise attached to the sale. Answer choice A is incorrect because the manufacturer waived the implied warranty of merchantability when he declined to inspect the machine. Answer choice B is incorrect because the seller did not cause the manufacturer’s unilateral mistake; rather, the seller offered to open the motor housing, and the manufacturer declined the offer. Answer choice C is incorrect because the warranty of merchantability is implied in all sales of goods and need not be expressly stated by the seller. If the defect could not have been uncovered by a reasonable inspection, the manufacturer would have had a valid claim against the seller for violation of the implied warranty of merchantability, even though the seller had not expressly made any such claims.

A supervisor and an employee in neighboring cubicles had been chatting about sports. Ten minutes after their sports discussion had ended, the supervisor offered to sell the employee tickets to an upcoming baseball game for $500, even though he saw the employee had placed earphones into his ears and begun working again. The employee, realizing that he missed something his supervisor said, nervously responded, “Sure, boss.” The day of the game, the supervisor told the employee that he would accept cash or check. When the employee indicated he had no idea what the supervisor was referencing, the supervisor angrily restated the terms of the discussion. Upon hearing the facts, the employee, who was short on funds, refused to pay the supervisor.
Has a contract between the supervisor and employee been formed?


A. Yes, because the supervisor reasonably believed the employee accepted the offer.B. Yes, because the employee accepted the terms of the contract.C. No, because a reasonable person would not believe the parties had entered into a contract.D. No, because the employee did not intend to accept the contract.

Answer choice C is correct. Contract formation is determined by an objective theory: whether reasonable people in the parties’ position, given the facts and circumstances, would conclude that a contract had been made based on objective manifestations of intent (regardless of actual subjective intent). In considering the circumstances surrounding the statement, it would not be reasonable to assume the parties entered a contract when the employee never even heard the offer. Answer choice A is incorrect because the subjective belief of a party is not the relevant consideration; instead, the court would consider the existence of the contract based on an objective theory. Answer choice B is incorrect because a reasonable person would not believe the employee intended to accept the terms of the offer, as he never even heard the offer. Answer choice D is incorrect because the intent to contract is generally determined by an objective theory, rather than a subjective one. While the earphones would indicate that the employee did not intend to be bound by the contract, it would be an objective standard that would determine the validity of the contract. While both answer choices C and D would likely arrive at the same conclusion—that there is no valid contract—the correct answer choice reflects that this would be based on an objective standard rather than a subjective one.

The owner of a restaurant who highlighted local ingredients when creating his menu bought cheese and other dairy products from a local dairy farmer. The owner and the farmer had entered into written requirements contracts each spring for the past ten years. In the winter of the tenth year, the farmer purchased a substantial amount of new dairy cows and expanded his farming capabilities. He notified all customers that he would have a much higher amount of available products the following spring. The owner responded with a date he wished for the products to be delivered, as per custom, but said nothing else. On the agreed upon date, the farmer delivered substantially more products than he would customarily provide. The owner attempted to accept half of the shipment, as that was roughly his customary quantity, but the farmer stated that the products were already packaged and that the owner should have spoken up after receiving the notice from the farmer. The owner then rejected the shipment in its entirety.
Did the owner breach the contract with the farmer as to this shipment?


A. No, because no contract existed, as the parties did not agree to a quantity.B. No, because the farmer made a nonconforming tender of goods.C. Yes, because the owner should have given the farmer time to cure the nonconformity.D. Yes, because the owner rejected the shipment in its entirety.

Answer choice B is correct. The parties here had a requirements contract, and the farmer delivered much more than the owner reasonably required, thereby making a nonconforming tender of goods. While some variation is permissible in requirements contracts, the farmer supplied double the amount of customary products, which is unreasonable, especially considering the shelf life of dairy products. Upon receiving a nonconforming tender of goods, the owner had the right to accept or reject all or part of shipment. Here, the owner tried first to accept half of the goods, and then rejected all of the nonconforming tender after the farmer did not allow him to do so. Answer choice A is incorrect because this is a requirements contract, under which a buyer agrees to buy all that he will require of a product from the other party. In a requirements contract under the UCC, any quantities under such a contract may not be unreasonably disproportionate to any stated estimates, or if no estimate is stated, to any normal or otherwise comparable prior requirements or output. Here, no quantity was stated, but the court could reference prior performance between the farmer and the owner. Answer choice C is incorrect because a seller only has a right to cure a nonconforming tender if the time for performance has not yet passed. Here, the time for performance was the date of the delivery, so the farmer did not have a right to cure; in any case, the farmer refused to tender a conforming amount when requested by the owner. Answer choice D is incorrect because the owner had the right to accept or reject part or all of the farmer’s nonconforming tender.

A farmer owned a tractor and offered his brother the chance to purchase it. He stated that his brother had to decide whether he wanted to purchase the tractor within “six months of today’s date.” The brother paid him $200 on that day to keep the option open. The agreement was reduced to writing, signed by both men, and dated May 15. The farmer died on July 1. On August 15, the brother notified the executor of the farmer’s estate that he wanted to accept the offer to buy the tractor. The executor refused to sell, and the brother filed suit for the enforcement of the contract.
Is the brother likely to prevail?


A. Yes, because the brother made an enforceable contract to buy the tractor on May 15.B. Yes, because the brother paid $200 to keep the option open.C. No, because the offer terminated on July 1.D. No, because at the time of the farmer’s death, the tractor went to his estate.

Answer choice B is correct. While an offer generally terminates upon the death or mental incapacity of the offeror, an exception exists for an offer that is an option. Valid options do not terminate upon death or mental incapacity of the offeror because consideration was paid to keep the offer open during the option period; the offer is therefore made irrevocable during that period. Here, the offer would not terminate upon the farmer’s death because it was an option for which consideration was paid, and it was still open at the time of the farmer’s death. Answer choice A is incorrect because on May 15, the parties made an option contract; the actual contract to buy the tractor was made on August 15. Answer choice C is incorrect because the offer did not terminate upon the farmer’s death; as explained above, it was an option contract. Answer choice D is incorrect. Although the tractor did go to the farmer’s estate, it is still subject to the option contract.

An honest dispute develops between a condominium owner and a plumber over whether plumbing installed in the kitchen and bathrooms of the condominium satisfied contractual specifications. If the plumbing meets those specifications, the condominium owner owes the plumber $15,000 under the terms of the contract. The condominium owner offers to pay the plumber $10,000 in satisfaction of the owner's contractual obligations, if the plumber replaces the plumbing in the kitchen with another grade of pipe. The plumber accepts the condominium owner's offer. After the plumber replaces the kitchen plumbing, the condominium owner refuses to pay the plumber. In a breach of contract action brought by the plumber, the fact-finder determines that the plumbing originally installed by the plumber did satisfy the contract specifications. The fact-finder also determines that the plumber and the condominium owner entered into a substitute agreement for which the owner failed to deliver the required performance.

What is maximum amount that the plumber can seek in damages from the condominium owner?


A. $25,000B. $15,000C. $10,000D. Nothing

Answer choice C is correct. Since the plumber and the condominium owner entered into a substitute agreement rather than an accord, the substitute agreement completely replaces the original contract. Consequently, the plumber may only seek damages of $10,000, the amount that the condominium owner promised to pay under that agreement. Had the second agreement between condominium owner and the plumber been an accord, the plumber could have sought damages upon the condominium owner's breach of the accord under either the original contract (i.e., $15,000) or the accord (i.e., $10,000), but not both. For this reason, answer choices A, B, and D are incorrect.

A daughter successfully petitioned a court to have her father declared incompetent to manage his affairs and to have herself appointed as guardian of his property. Subsequently, the father ordered furniture totaling $3,500 from a local store. The store, unaware of the guardianship and not otherwise having a reason from the father’s behavior to learn of his incompetency, delivered the furniture to the father’s residence where he received and accepted it. The next day a flood destroyed the furniture before the daughter had the opportunity to contact the store.

Is the store entitled to enforce the contract for the sale of the furniture?


A. Yes, because the risk of loss had passed to the father, as buyer of the furniture, upon its delivery.B. Yes, because the store was unaware of the guardianship and the father’s incompetency.C. No, because the daughter did not have the opportunity to contact the store.D. No, because the father had been adjudicated incompetent.

Answer choice D is correct. An individual who is the subject of a court-ordered guardianship over that individual’s property lacks the capacity to enter into a contract. Consequently, any contract purportedly entered into by such an individual is void. Since the father was under a court-ordered guardianship, his contract to purchase the furniture was void. (Note: If the furniture is a necessity, the store may be able to recover the furniture’s reasonable value in restitution under a quasi-contract action, but cannot enforce the contract of sale between the store and the father.)

When a grandmother's 17-year-old granddaughter finished high school, the grandmother handed her a signed memorandum stating that if she would go to college for four academic years, she would pay her tuition, room, and board and would "give her a $1,000 bonus" for each "A" she got in college. The granddaughter's uncle, who was present on this occasion, read the memorandum and thereupon said to the granddaughter, "and if Grandma doesn't pay your expenses, I will." The grandmother paid her tuition, room, and board for her first year but died just before the end of that year. Subsequently, the granddaughter learned that she had received two "A's" in the second semester. The executor of the grandmother's estate has refused to pay her anything for the two "A's" and has told her that the estate will no longer pay her tuition, room, and board in college.

In an action against the grandmother's estate for $2,000 on account of the two "A's" if the only defense raised is lack of consideration, the granddaughter probably will


A. succeed under the doctrine of promissory estoppel.B. succeed on a theory of bargained-for exchange for her grandmother's promise.C. not succeed, because the $1,000 for each "A" was promised only as a bonus.D. not succeed, because the granddaughter was already legally obligated to use her best efforts in college.

Answer choice B is correct. The general rule is that in addition to an offer and acceptance, some legal detriment must be bargained for in order for a contract to exist. Here, the granddaughter can successfully argue that her acceptance of her grandmother's unilateral offer was evidence by performance, through the two "A's". Answer choice A is incorrect because promissory estoppel is only available when a contract was not formed; here, a unilateral contract was formed.

A debtor's $1,000 contractual obligation to a lender was due on July 1. On June 15, the lender called her niece and said, "As my birthday gift to you, you may collect on July 1 the $1,000 that the debtor owes me." The lender also called the debtor and told him to pay the $1,000 to the niece on July 1. On July 1, the debtor, saying that he did not like the niece and wouldn't pay anything to her, paid the $1,000 to the lender, who accepted it without objection.

Will the niece succeed in an action for $1,000 against the debtor?


A. Yes, because the lender had effectively assigned the $1,000 debt to her.B. Yes, because the lender's calls to the niece and the debtor effected a novation.C. No, because the lender's acceptance of the $1,000, without objection, was, in effect, the revocation of a gratuitous assignment.D. No, because the debtor cannot be compelled to render performance to an assignee whom he finds personally objectionable.

Answer choice C is correct. If no consideration supports the assignment (a gratuitous assignment), it will generally be revocable, unless the obligor has already performed or promissory estoppel applies. In this case, the obligor had not performed, and there is no basis under these facts for the niece to claim reliance sufficient to support a claim of promissory estoppel. Answer choice A is incorrect, as the assignment to the niece was a gratuitous assignment because there was no consideration given by the niece, and it could therefore be revoked by the lender. Answer choice B is incorrect, as there was no novation in this case. A novation is the substitution of a new contract for an old one, where the original obligor is released from his promises under the original agreement. Answer choice D is incorrect, as a debtor may be so compelled if the assignment is valid. Here, though, the assignment was gratuitous and could be revoked.


An adult woman bought a rare baseball card from a seller and promised to pay $100 "as soon as I am able."

What effect does this quoted language have on enforceability of the promise?


A. None.B. It makes the promise illusory.C. It requires the woman to prove her inability to pay.D. It requires the seller to prove the woman's ability to pay.

Answer choice D is correct. The quoted language is a condition precedent to the woman's obligation to pay. If the condition is precedent, the plaintiff has the burden of proving that the condition occurred in order to recover. Answer choice A is incorrect, as the quoted language is a condition precedent, which would put the burden of proof on the seller to prove the woman's ability to pay. Answer choice B is incorrect. An illusory promise is one that essentially pledges nothing because it is vague or because the promisor can choose whether or not to honor it. Here, the promise is not vague; it is simply conditional on the woman having the ability to pay. Answer choice C is incorrect, as this is a condition precedent, which places the burden on the seller (as plaintiff) to prove the woman's ability to pay. The foregoing NCBE MBE question has been modified to reflect current NCBE stylistic approaches; the NCBE has not reviewed or endorsed this modification.

A buyer bought 100 reams of No. 1 linen paper, in snow white, from a seller. The sales contract provided that the buyer would make payment prior to inspection. The 100 reams were shipped, and the buyer paid the seller. Upon inspection, however, the buyer discovered that the paper was tundra white, a slightly darker shade of white than snow white. The buyer thereupon tendered back the paper to the seller and demanded return of his payment. The seller refused on the ground that there is no practical difference between tundra white and snow white paper.

What is the buyer's remedy because the paper was nonconforming?


A. Specific performanceB. Damages measured by the difference between the value of the goods delivered and the value of conforming goodsC. Damages measured by the price paid plus the difference between the contract price and the cost of buying substitute goodsD. None, since he waived his remedies by agreeing to pay before inspection

Answer choice C is correct. Under the UCC, which governs the sales of goods, damages for failing to deliver goods is measured by the difference between the contract price and the market value of the goods (or the cost of cover). Answer choice D is incorrect because the seller is not allowed to deliver nonconforming good without recourse by the buyer; when a buyer must pay for goods prior to inspection, as is the case here, the buyer is entitled to inspect the goods and revoke acceptance upon finding that the goods are nonconforming. Answer choice A is incorrect because specific performance is generally not granted for goods which are not unique; here, it is unlikely that No. 1 linen paper in snow white is a unique good. Answer choice B misstates the calculation of value of the goods and the measure of damages. The correct measure is the price paid for the nonconforming paper plus the difference between the contract price and the cost of buying substitute goods.


An adult daughter called a local restaurant to place a large delivery order. The restaurant generally requires a credit card for all delivery orders, but the daughter’s father, who is a regular at the restaurant and happened to be there when the daughter placed the order, told the clerk that, in the event the daughter failed to pay for the food, he would do so. The restaurant delivered the order to the daughter, who, having decided to order something else instead, refused to accept or pay for the food.

Can the restaurant collect from the father?


A. No, because the father’s promise was made orally.B. No, because a third party will not be held liable for the contract obligations of another.C. Yes, because the father promised to pay.D. Yes, because a parent is liable to pay for necessities provided to a child.


Incorrect: Answer choice A is correct. The Statute of Frauds applies to suretyship agreements (i.e., one person’s promise to pay the debts of another). Here, the daughter ordered food and was obligated to pay for the food. However, the father also promised to pay for the food if the daughter did not. This promise created a suretyship agreement. Since this agreement was not in writing, the restaurant cannot enforce it. Note that some oral suretyship contracts can be enforced (indemnity contracts and contracts wherein the surety’s main reason for paying the debt is the surety’s own economic advantage), but those circumstances are not present here.

-

On January 2, a builder and a luxury fashion designer entered into a written contract in which the builder agreed to build a boutique storefront for the designer, according to plans and specifications furnished by the designer's architects, at a contract price of $200,000. The contract provided for specified progress payments and a final payment of $40,000 upon the designer's acceptance of the boutique space and issuance of a certificate of final approval by the architect. Further, under a "liquidated damages" clause in the agreement, the builder promised to pay the designer $500 for each day's delay in completing the boutique after the following October 1. The designer, however, told the builder on January 2, before the contract was signed, that the designer would be out of town in Paris most of the summer and fall and would not return to open the boutique until November 1. Because the builder was overextended on other construction jobs, the builder did not complete the boutique until October 15. The designer returned on November 1 as planned and opened the boutique. Ten days later, after making the $40,000 final payment to the builder, the designer learned for the first time that the boutique had not been completed until October 15.

If the designer sues the builder for breach of contract on account of the fifteen-day delay in completion, which of the following will the court probably decide?


A. The designer will recover damages as specified in the contract, i.e., $500 multiplied by fifteen. B. The designer will recover the actual damages, if any, caused by the delay in completion.C. Having waived the delay by opening the boutique and making the final payment, the designer will recover nothing.D. The designer will recover nothing because the contractual completion date was impliedly modified to November 1 when the designer advised the builder on January 2 about the designer's Paris trip and return date.

Answer choice A is correct. Once a duty to perform exists, nonperformance is a breach of contract unless the duty is discharged. Here the date of completion was an express condition, by which the builder is subject to a measure of damages, if any, by the designer. Answer choice B is incorrect because the parties included a liquidated damages clause in the contract; so long as the liquidated damages do not constitute a penalty, they will be awarded over other contractual remedies. Answer choice C is incorrect because the designer did not waive rights to liquidated damages by opening the boutique and making the final payment. Answer choice D is incorrect because the liquidated damages provision is applicable regardless of whether the builder knew of the designer's trip or not. The foregoing NCBE MBE question has been modified to reflect current NCBE stylistic approaches; the NCBE has not reviewed or endorsed this modification.

A series of arsons, one of which damaged a store, occurred in a city. Subsequently, the damaged store, by a written memorandum to a private detective, proposed to pay the detective $200 "for each day's work you actually perform in investigating our fire." Following several months of investigation by the detective, an employee of the damaged store voluntarily confessed to the detective to having committed the arson of the store. The damaged store's president thereupon paid the detective at the proposed daily rate for his investigation.

Which of the following best characterizes the relationship between the damaged store and the detective?


A. A unilateral offer of employment by the damaged store which became irrevocable for a reasonable number of days after the detective commenced his investigation of the store's arson.B. An employment for compensation subject to a condition precedent that the detective succeed in his investigation.C. A series of daily bilateral contracts, the store exchanging an express promise to pay the daily rate for the detective's implied promise to pursue his investigation with reasonable diligence.D. A series of daily unilateral contracts, the store exchanging an express promise to pay the daily rate for the detective's daily activity of investigating the store's arson.

Answer choice D is correct. A unilateral contract is one where one party promises to do something in return for an act of the other party. Unlike a bilateral contract, the offeree's promise to perform is insufficient to constitute acceptance; the offeree must perform the act to accept the offer. Similarly, the detective's employment was a series of unilateral contracts based on the damaged store's offer. Answer choice A is incorrect because while the contract was unilateral, it never became irrevocable because of the consistency or persistence of the detective's activity. Answer choice B is incorrect because this is not an employment contract. Answer choice C is incorrect because bilateral contracts are formed when the offeror invites acceptance by a return promise, not by an act, which is the case here. The foregoing NCBE MBE question has been modified to reflect current NCBE stylistic approaches; the NCBE has not reviewed or endorsed this modification.

On June 15, a teacher accepted a contract for a one-year position teaching math at a public high school at a salary of $50,000, starting in September. On June 22, the school informed the teacher that, due to a change in its planned math curriculum, it no longer needed a full-time math teacher. The school offered instead to employ the teacher as a part-time academic counselor at a salary of $20,000, starting in September. The teacher refused the school's offer. On June 29, the teacher was offered a one-year position to teach math at a nearby private academy for $47,000, starting in September. The teacher, however, decided to spend the year completing work on a graduate degree in mathematics and declined the academy's offer.
If the teacher sues the school for breach of contract, what is her most likely recovery?


A. $50,000, the full contract amount.B. $30,000, the full contract amount less the amount the teacher could have earned in the counselor position offered by the school.C. $3,000, the full contract amount less the amount the teacher could have earned in the teaching position at the academy.D. Nothing, because the school notified the teacher of its decision before the teacher had acted in substantial reliance on the contract.

Answer choice C is correct. The teacher is entitled to recover damages that will place her in the position she would have been in but for the school's breach. However, an injured party is expected to make reasonable efforts to mitigate the loss resulting from the other party's breach. In the case of a wrongfully discharged employee, the employee is expected to accept an offer of comparable employment. If the employee fails or refuses to do so, the employee's recovery is reduced by the amount of the loss that the employee could have avoided by accepting comparable employment. Here, the teacher's damages of $50,000 should be reduced by the $47,000 she would have earned if she had accepted the comparable teaching position at the private academy. Therefore, the teacher is entitled to recover $3,000 from the school.

While negligently driving his father's uninsured automobile, a 25-year-old son crashed into an automobile driven by a woman. Both the son and the woman were injured. The father, erroneously believing that he was liable because he owned the automobile, said to the woman: "I will see to it that you are reimbursed for any losses you incur as a result of the accident." The father also called a physician and told him to take care of the woman, and that he, the father, would pay the bill.

Which of the following, if true, would be significant in determining whether or not there was bargained-for consideration to support the father's promise to the physician?


A. The physician had not begun treating the woman before the father called him.B. The father was estranged from his son.C. The woman was unemployed and did not have insurance.D. The physician was a private doctor.


Correct: Answer choice A is correct. For the doctor to assert that he has a contract with the father there must be a bargained-for exchange. If the doctor had already treated the woman, there is no exchange and therefore no consideration. If the doctor treats the woman after the call, there is an exchange and there is consideration. The answer choices B, C, and D have no bearing on whether there is bargained-for consideration. Thus, answer choice A is the best answer.


-

A lawyer and a landscape architect signed a detailed writing in which the landscape architect agreed to landscape a tract of land in front of the lawyer's office in accordance with a design prepared by the landscape architect and incorporated in the writing. The lawyer agreed to pay $15,000 for the work upon its completion. At the lawyer's insistence, the written lawyer-landscape architect agreement contained a provision that neither party would be bound unless the lawyer's paralegal, an avid student of landscaping, should approve the landscape architect's design. Before the landscape architect commenced the work, the lawyer's paralegal, in the presence of both the lawyer and the landscape architect, expressly disapproved the landscaping design. Nevertheless, the lawyer ordered the landscape architect to proceed with the work, and the landscape architect reluctantly did so. When the landscape architect's performance was 40 percent complete, the lawyer repudiated her duty, if any, to pay the contract price or any part thereof.

If the landscape architect now sues the lawyer for damages for breach of contract, which of the following concepts best supports the landscape architect's claim?


A. Substantial performanceB. Promissory estoppelC. Irrevocable waiver of conditionD. Unjust enrichment

Answer choice C is correct. A party whose duty is subject to a condition can waive the condition, either by words or conduct. In general, a waiver operates to preclude a subsequent assertion of the right waived or any claim based on such right, and will be irrevocable once made, even in the absence of consideration, or of any change in position of the party in whose favor the waiver operates. Here, the lawyer specifically ordered the landscape architect to proceed with the work, waiving her contractual right to avoid the contract.

A florist entered into a contract with a builder by the terms of which the builder was to renovate the florist's greenhouse for $8,000 and was required to do all of the work in one week, starting the following Saturday, so as to avoid disruption of the florist's business. The builder commenced work on Saturday morning, and had finished half the renovation by the time he quit work for the day. That night, without the fault of either party, the greenhouse was destroyed by fire.

Which of the following is an accurate statement?


A. Both parties' contractual duties are discharged, and the builder can recover nothing from the florist.B. Both parties' contractual duties are discharged, but the builder can recover on quasi-contract from the florist.C. Only the builder's contractual duty is discharged, because the florist's estimated performance (payment of the agreed job price) is not impossible.D. Only the builder's contractual duty is discharged, and the builder can recover his reliance damages from the florist.

Answer choice B is correct. The specific subject matter of the contract, the greenhouse, was destroyed through no fault of either party, making the contract impossible to discharge. A party may recover in quasi-contract for any benefit that was conferred prior to impossibility, however. Answer choice A is incorrect, as quasi-contractual recovery is permitted for any benefit conferred prior to impossibility. Answer choice C is incorrect, as the impossibility defense applies to discharge both the builder's and the florist's contractual duties. Answer choice D is incorrect, as both parties' duties are discharged by impossibility, not merely the builder's. The foregoing NCBE MBE question has been modified to reflect current NCBE stylistic approaches; the NCBE has not reviewed or endorsed this modification.

A salon owner contacted a manufacturer by email about purchasing shampoo sinks. The manufacturer sent the salon owner the following email: “I will sell you four shampoo sinks at a discounted price of $300 apiece.” The salon owner responded immediately, rejecting the offer. However, due to a transmission problem in the internet routing system, the message was not delivered to the manufacturer until the following day. In the meantime, the salon owner contacted several other sellers, all of whom made significantly higher offers. The salon owner then sent another email to the manufacturer, stating, “I accept your offer.” This email was delivered immediately. The following day, the misrouted rejection email arrived in the manufacturer’s inbox. Assume the parties are in a jurisdiction that applies the mailbox rule to electronic communications.
Was a contract formed?


A. Yes, because the salon owner accepted the manufacturer’s offer.B. Yes, because the mailbox rule applies.C. No, because the salon owner rejected the offer prior to accepting the offer.D. No, because the manufacturer received the salon owner’s rejection.

Answer choice A is correct. The mailbox rule states that a timely sent acceptance is effective upon posting, not upon receipt. However, if a communication is sent rejecting the offer, and a later communication is sent accepting the contract, the mailbox rule does not apply, and the first one to be received by the offeror prevails. Here, the salon owner rejected the offer, then sent an acceptance. Since the acceptance was received by the offeror first, the acceptance prevails. Answer choice B is incorrect because, as noted with respect to answer choice A, the mailbox rule does not apply when an acceptance is sent after a rejection. Answer choice C is incorrect because, while a rejection’s effect is to terminate an offer, to be effective, the rejection must be communicated to the offeror. When a rejection is sent before an acceptance, the mailbox rule does not apply and the first communication received by the offeror prevails. Answer choice D is incorrect because a contract was formed when the manufacturer received the acceptance before the rejection. The fact that the manufacturer received the rejection the following day may affect the damages to which the manufacturer is entitled if the salon owner refuses to honor the

A charity, seeking to raise funds, held a legally permitted raffle in which the prize is a new automobile. A week before the raffle, the organizer of the raffle contacted a friend who had purchased a raffle ticket. The organizer promised to ensure that the friend would win the raffle if the friend gave the organizer $1,000. The friend agreed and gave the organizer $1,000. On the day before the raffle, the friend began to feel guilty. He went to the organizer, renounced the scheme, and demanded his $1,000 back. The organizer refused. The next day at the raffle, the automobile was awarded to someone else.

Can the friend successfully bring a legal action against the organizer?


A. No, because the agreement between the friend and the organizer was illegal.B. No, because the friend failed to take any action to prevent the raffle from being held.C. Yes, because there was a valid contract between the organizer and the friend.D. Yes, because the friend is entitled to a return of the $1,000 paid to the organizer.

Answer choice D is correct. Although the friend is not entitled to enforce the illegal agreement between him and the organizer, the friend is entitled to restitution with regard to the $1,000 paid to the organizer since he withdrew from the transaction before the raffle was held and did not engage in serious misconduct. For this reason, answer choice A is incorrect. Answer choice B is incorrect because, even though the friend’s failure to take action to prevent the raffle from being held may result in criminal liability, the friend’s action in withdrawing from the transaction before the illegal purpose was accomplished does give rise to the right to restitution. Answer choice C is incorrect because, although there was a valid offer and acceptance and valuable consideration was given by both the friend and the organizer, the contract was made for an illegal purpose and therefore is unenforceable.

On March 1, a homeowner contacted a builder about constructing an addition to the homeowner's house. The builder orally offered to perform the work for $200,000 if his pending bid on another project was rejected. The homeowner accepted the builder's terms and the builder then prepared a written contract that both parties signed. The contract did not refer to the builder's pending bid. One week later, upon learning that his pending bid on the other project had been accepted, the builder refused to perform any work for the homeowner.

Can the homeowner recover for the builder's nonperformance?


A. No, because efficiency principles justify the builder's services being directed to a higher-valued use.B. No, because the builder's duty to perform was subject to a condition.C. Yes, because the builder's attempt to condition his duty to perform rendered the contract illusory.D. Yes, because the parol evidence rule would bar the builder from presenting evidence of oral understandings not included in the final writing.

Answer choice B is correct. The condition exception to the parol evidence rule permits the admission of extrinsic evidence to establish an oral condition to the parties' performance under the contract. Restatement (Second) of Contracts § 217. Although contract law supports the principle of efficient breach, the principle does not discharge a breaching party's duty to perform or its responsibility to compensate the non-breaching party for damages arising from the breaching party's failure to perform. Restatement (Second) of Contracts § 346. As such, answer choice A is incorrect. Answer choice C is incorrect because the builder made a definite commitment to perform even though it was conditioned on his other bid being rejected. The builder's promise to perform was not illusory since the possibility the condition might occur limited the builder's freedom of action by binding him to perform if the condition was satisfied. Restatement (Second) of Contracts § 76. Answer choice D is incorrect based on the rationale stated in correct answer B.

Two men were tennis acquaintances at a country club. Both traveled for work: one for a food distribution company and one for an insurance company. The food distributor wrote the insurance agent by United States mail on Friday, April 24: I need a scooter for transportation to the country club, and will buy your ZVR scooter for $1,000 upon your bringing it to my home address above [stated in the letterhead] on or before noon, May 12 next. This offer is not subject to countermand. Sincerely, [signed] [the food distributor]. The insurance agent replied by mail the following day: I accept your offer, and promise to deliver the scooter as you specified. Sincerely, [signed] [the insurance agent]. This letter, although properly addressed, was misdirected by the postal service and not received by the food distributor until May 10. The food distributor had bought another ZVR scooter from a baker he met at the country club for $850 a few hours before. The baker saw the insurance agent at the country club on May 11 and said: "I sold my ZVR to the food distributor yesterday for $850. Would you consider selling me yours for $750?" The insurance agent replied: "I'll let you know in a few days." On May 12, the insurance agent took his ZVR scooter to the food distributor's residence; he arrived at 11:15 a.m. The food distributor was asleep and did not answer the insurance agent's doorbell rings until 12:15 p.m. The food distributor then rejected the insurance agent's scooter on the ground that he had already bought the baker's.


What is the probable legal effect of the baker's conversation with the insurance agent and report that he (the baker) had sold his ZVR to the food distributor on May 10?


A. This report had no legal effect because the food distributor's offer was irrevocable until May 12.B. Unless a contract had already been formed between the insurance agent and the food distributor, the baker's report to the insurance agent operated to terminate the insurance agent's power of accepting the food distributor's offer.C. This report has no legal effect because the offer had been made by a prospective buyer (the food distributor) rather than a prospective seller.D. The baker's conversation with the agent on May 11 terminated the food distributor's original offer and operated as an offer by the baker to buy the insurance agent's ZVR scooter for $750.


Answer choice B is correct. An offer can only be accepted when it is still outstanding, but it may be terminated by the offeror at any time, even if the offer states that it will remain open for a specified period of time. Thus answer choice A is incorrect. A revocation is not effective until communicated, but if an offeree acquires reliable information that the offeror has taken definite action inconsistent with the offer, then the offer is automatically revoked. Here, once the insurance agent became aware of the food distributor's revocation, he had no power to accept the original offer. Answer choice C is incorrect because the baker's report did have legal effect: to notify the insurance agent that the baker had sold his ZVR scooter to the food distributor, thereby automatically revoking the original offer to the insurance agent. Answer choice D is incorrect because it is unclear from the baker's language whether his question is an inquiry or an actual offer to buy the insurance agent's ZVR scooter. The foregoing NCBE MBE question has been modified to reflect current NCBE stylistic approaches; the NCBE has not reviewed or endorsed this modification.

For an agreed price of $20 million, a builder contracted with a company to design and build on the company's commercial plot a 15-story office building. In excavating for the foundation and underground utilities, the builder encountered a massive layer of granite at a depth of 15 feet. By reasonable safety criteria, the building's foundation required a minimum excavation of 25 feet. When the contract was made, neither the company nor the builder was aware of the subsurface granite, for the presence of which neither party had hired a qualified expert to test. Claiming accurately that removal of enough granite to permit the construction as planned would cost him an additional $3 million and a probable net loss on the contract of $2 million, the builder refused to proceed with the work unless the company would promise to pay an additional $2.5 million for the completed building.

If the company refuses and sues the builder for breach of contract, which of the following will the court probably decide?


A. The builder is excused under the modern doctrine of supervening impossibility, which includes severe impracticability.B. The builder is excused because the contract is voidable on account of the parties' mutual mistake concerning an essential underlying fact.C. The company prevails because the builder assumed the risk of encountering subsurface granite that was unknown to the company.D. The company prevails, unless subsurface granite was previously unknown anywhere in the vicinity of the company's construction site.


Answer choice C is correct. While neither party was aware of the existence of the granite, the possibility of encountering subsurface granite in an excavation is not so unusual that the non-occurrence of finding it could be found to be a basic assumption of the contract. The builder therefore assumed the risk that such granite would be found when he accepted the contract. Answer choice A is incorrect, as supervening impossibility only applies where an event occurs and the non-occurrence of which was a basic assumption on which the contract was made. Here, there is no indication that the non-existence of subsurface granite was a basic assumption of the parties. Answer choice B is incorrect. Mutual mistake occurs when both parties are mistaken as to an essential element of the contract. Here there was no "mistake." The parties simply did not consider what might be under the surface of the site. Answer choice D is incorrect, as even if subsurface granite was previously unknown anywhere in the vicinity of the company's site, the builder still assumed the risk of encountering subsurface granite that was unknown to the company.

An experienced rancher contracted to harvest his neighbor's wheat crop for $1,000 "when the crop [was] ripe." In early September, the neighbor told the rancher that the crop was ripe. The rancher delayed because he had other customers to attend to. The neighbor was concerned that the delay might cause the crop to be lost, for hailstorms were common in that part of the country in the fall. In fact, in early October, before the crop was harvested, it was destroyed by a hailstorm.

Is the rancher liable for the loss?


A. No, because no time for performance was established in the contract.B. No, because the neighbor failed to tell the rancher that the crop might be destroyed by a hailstorm.C. Yes, because at the time the contract was made, the rancher had reason to foresee the loss as a probable result of his breach.D. Yes, because a party who undertakes a contractual obligation is liable for all the consequences that flow from his breach.

Answer choice C is correct. According to the Restatement (Second) of Contracts, a non-breaching party is entitled to recover damages that the party in breach "had reason to foresee as a probable result of the breach" when the parties entered into the contract. The rancher's experience and the frequency of hailstorms in the fall combined to make the loss resulting from the rancher's breach foreseeable. Answer choice A is incorrect because the language in the contract tying the rancher's performance to when the crop was ripe was sufficient to establish the time of the rancher's performance. The rancher will be liable for the loss because such a loss was foreseeable. Answer choice B is incorrect. According to the Restatement (Second) of Contracts, reason to foresee can arise from circumstances that result in the breaching party having had actual or constructive knowledge of the loss that might result from the breach. Therefore, the neighbor's failure to specifically inform the rancher that the crop might be destroyed by a hailstorm does not determine whether the loss was foreseeable. The rancher's experience and the frequency of hailstorms in the fall combined to make the loss resulting from the rancher's breach foreseeable. Answer choice D is incorrect because the rules of contract law limit the recoverability of damages for breach. In this case, the rancher will be liable for the loss because, as previously explained, such a loss was foreseeable.

The aged owner of a mansion wished to employ a live-in companion so that she might continue to live in her own home. The owner, however, had only enough income to pay one half of the companion's $4,000 monthly salary. Learning of their beloved employer's plight, the gardener and the owner's personal chef, both adults, agreed with each other in a signed writing that, on the last day of March and each succeeding month during the owner's lifetime, each would give the owner $1,000. The owner then hired the companion. The gardener and chef made the agreed payments in March, April, and May. In June, however, the chef refused to make any payment and notified the gardener and the owner that he would make no further payments.

Will the owner succeed in an action for $1,000 brought against the chef after June 30?


A. Yes, because by making his first three payments, the chef confirmed his intent to contract.B. Yes, because the owner is an intended beneficiary of a contract between the gardener and the chef.C. No, because an employer cannot sue her employee for breach of a promise for support.D. No, because the gardener and the chef intended their payments to the owner to be gifts.

Answer choice B is correct. An intended beneficiary is one to whom the promisee wishes to make a gift of the promised performance. The promisee must have an intention (explicit or implicit) to benefit the third party. An intended beneficiary has a right to bring an action on the contract. An intended beneficiary of a gift promise may sue only the promisor. If the promisee tells the donee beneficiary of the contract and should reasonably foresee reliance, and the beneficiary does so rely to her detriment, promissory estoppel would apply. Here, the owner is an intended beneficiary, as the gardener and chef intended to benefit her by their contract. Answer choice A is incorrect, as it is irrelevant whether the chef made the first three payments. The owner is an intended beneficiary of the contract and entitled to recover from the chef for June, whether or not he paid in March, April, or May. Answer choice C is incorrect, as an employer can sue an employee for breach of promise for support. Answer choice D is incorrect, as even if intended to be gifts, the owner is entitled to sue as a donee intended third-party beneficiary. The foregoing NCBE MBE question has been modified to reflect current NCBE stylistic approaches; the NCBE has not reviewed or endorsed this modification.

A written contract was entered into between a venture capitalist and a mustard seed farmer. The contract provided that the venture capitalist would invest $750,000 for capital expansion of the farmer’s farming and production facilities and, in return, that the farm would produce and market at least 500,000 jars of artisanal Dijon-style mustard each year for five years under the label "Taste of Paris." The contract included provisions that the parties would share equally the profits and losses from the venture and that, if feasible, the mustard would be distributed only through a particular wholesale distributor of fine foods. Neither the venture capitalist nor the farmer had previously dealt with the distributor. The distributor learned of the contract two days later from reading a trade newspaper. In reliance thereon, he immediately hired an additional sales executive and contracted for enlargement of his warehousing and display facility. Soon after making the contract with the venture capitalist, the farmer, without the venture capitalist’s knowledge or assent, sold his farmland but not his production facilities to a large agricultural corporation. Under the terms of this sale, the corporation agreed to sell to the farm all mustard seeds grown on the land for five years. The corporation's employees have no experience in mustard seed production, and the corporation has no reputation in the food industry as a mustard producer or otherwise. The venture capitalist-farmer contract was silent on the matter of the farm selling any or all of its business assets.

If the venture capitalist seeks an appropriate judicial remedy against the farmer for entering into the farm-corporation transaction, is the venture capitalist likely to prevail?


A. Yes, because the farm-corporation transaction created a significant risk of diminishing the profits in which the venture capitalist would share under his contract with the farm.B. Yes, because the venture capitalist-farm contract did not contain a provision authorizing a delegation of the farm's duties.C. No, because the farm remains in a position to perform under the venture capitalist-farm contract.D. No, because the farm, as a corporation, must necessarily perform its contracts by delegating duties to individuals.

Answer choice A is correct. Assignments are not allowed when doing so materially increases the duty or risk of the obligor or materially reduces the obligor's chance of obtaining performance. Answer choices C and D are incorrect because both answers ignore the potentially negative effect that the farm-corporation transaction had on the original relationship and the fact that the assignment would materially increase the venture capitalist's exposure to risk. Answer choice B is incorrect because generally, prohibitions against assignment in the contract are strictly construed, and the assignment is permissible. The foregoing NCBE MBE question has been modified to reflect current NCBE stylistic approaches; the NCBE has not reviewed or endorsed this modification.

A radio manufacturer and a retailer, after extensive negotiations, entered into a final, written agreement in which the manufacturer agreed to sell and the retailer agreed to buy all of its requirements of radios, estimated at 20 units per month, during the period January 1, 2007, through December 31, 2009, at a price of $50 per unit. A dispute arose in late December 2009 when the retailer returned 25 non-defective radios to the manufacturer for full credit after the manufacturer had refused to extend the contract for a second three-year period. In an action by the manufacturer against the retailer for damages due to return of the 25 radios, the manufacturer introduces the written agreement, which expressly permitted the buyer to return defective radios for credit but was silent as to return of non-defective radios for credit. The retailer seeks to introduce evidence that, during the three years of the agreement, it had returned, for various reasons, 125 non-defective radios, for which the manufacturer had granted full credit. The manufacturer objects to the admissibility of this evidence. When the retailer returned the 25 radios in question, it included with the shipment a check payable to the manufacturer for the balance admittedly due on all other merchandise sold and delivered to the retailer. The check was conspicuously marked, "payment in full for all goods sold to the retailer to date." The manufacturer's credit manager, reading this check notation and knowing that the retailer had also returned the 25 radios for full credit, deposited the check without protest in the manufacturer's local bank account. The canceled check was returned to the retailer a week later.

Which of the following defenses would best serve the retailer?


A. The manufacturer's deposit of the check and its return to the retailer after payment estopped the manufacturer thereafter to assert that the retailer owed any additional amount.B. By depositing the check without protest and with knowledge of its wording, the manufacturer discharged any remaining duty to pay on the part of the retailer.C. By depositing the check without protest and with knowledge of its wording, the manufacturer entered into a novation discharging any remaining duty to pay on the part of the retailer.D. The parties' good-faith dispute over return of the radios suspended the duty of the retailer, if any, to pay any balance due.

Answer choice B is correct. This is considered an "accord and satisfaction," which discharges both the original contract and the accord contract. There was a dispute about the radios, and the manufacturer's deposit of the check with the notation discharges any duties of the retailer. Answer choice A is incorrect, as this is not an estoppel issue, but an accord and satisfaction, which discharges any contractual duties of the retailer. Answer choice C is incorrect, as this is not a novation, but an accord and satisfaction. Answer choice D is incorrect, as the good-faith dispute did not suspend the duty of the retailer. The accord and satisfaction discharged both the original contract and the accord contract.

A baseball star contracted with a municipal symphony orchestra to perform for $5,000 at a children's concert as narrator of "Peter and the Wolf." Shortly before the concert, the baseball star became embroiled in a highly publicized controversy over whether he had cursed and assaulted a baseball fan. The orchestra canceled the contract out of concern that attendance might be adversely affected by the baseball star's appearance. The baseball star sued the orchestra for breach of contract. His business agent testified without contradiction that the cancellation had resulted in the star's not getting other contracts for performances and endorsements. The trial court instructed the jury, in part, as follows: "If you find for the plaintiff, you may award damages for losses which, at the time of contracting, could reasonably have been foreseen by the defendant as a probable result of its breach. However, the law does not permit recovery for the loss of prospective profits of a new business caused by breach of contract."

On the baseball star's appeal from a jury verdict in his favor, and judgment thereon, awarding damages only for the $5,000 fee promised by the orchestra, the judgment will probably be


A. affirmed, because the trial court stated the law correctly.B. affirmed, because the issue of damages for breach of contract was solely a jury question.C. reversed, because the test for limiting damages is what the breaching party reasonably could have foreseen at the time of the breach.D. reversed, because under the prevailing modern view, lost profits of a new business are recoverable if they are established with reasonable certainty.

Answer choice D is correct. The instructions to the jury were incorrect because the modern trend is to allow recovery for lost profits that are established with reasonable certainty. The testimony of the baseball star's business agent may meet the test of reasonably certainty, so the judgment will be reversed. Answer choice A is incorrect, as the trial court did not state the law correctly. Answer choice B is incorrect because, although it is a true statement that the issue of damages is for the jury to decide, the judgment must be reversed if the jury instructions were improper. Although the conclusion of answer choice C is correct, the reasoning is not correct because, if the jury instruction had been consistent with the modern trend, then the jury's decision would have been upheld. It is only because the instruction was improper that the judgment will be reversed.

A company, in a signed writing, contracted with a mechanic, for the sale to the buyer of 70 identical sets of specified wrenches, 25 sets to be delivered on April 15, and the remaining 45 sets on May 15. The agreement did not specify the place of delivery, or the time or place of payment.

Which of the following statements is correct?


A. The company must tender 25 sets to the mechanic at the mechanic's place of business on April 15, but does not have to turn them over to the mechanic until the mechanic pays the contract price for the 25 sets.B. The company has no duty to deliver the 25 sets on April 15 at the company's place of business unless the mechanic tenders the contract price for the 25 sets of wrenches on that date.C. The company must deliver 25 sets on April 15, and the mechanic must pay the contract price for the 25 sets within a reasonable time after their delivery.D. The company must deliver 25 sets on April 15, but the mechanic's payment is due only upon the delivery of all 70 sets.

Answer choice B is correct. This agreement is for an installment contract because the goods are to be delivered in a number of shipments. When the contract is silent as to payment due date, the U.C.C. gap-filler provisions make payment due upon each delivery (unless the price cannot be apportioned). Answer choice A is incorrect because, when a contract is silent as to place of delivery, the U.C.C. gap-filler default is the seller's place of business. Answer choice C is incorrect, as here the payments can be apportioned and are due upon each delivery according to the U.C.C. Answer choice D is incorrect because here, the price can be apportioned and thus, the price is due upon each delivery. The foregoing NCBE MBE question has been modified to reflect current NCBE stylistic approaches; the NCBE has not reviewed or endorsed this modification.


A buyer contracted in writing with an employee of DZL Corporation, who owned all of DZL Corporation's fixtures, to purchase all of her fixtures at a specified price per item. At the time this contract was executed, the buyer's contracting officer said to the employee, "Of course, our commitment to buy is conditioned on our obtaining approval of the contract from our parent company." The employee replied, "Fine. No problem." The parent company orally approved the contract, but the employee changed her mind and refused to consummate the sale on two grounds: (1) when the agreement was made, there was no consideration for her promise to sell; and (2) the parent company's approval of the contract was invalid.

If the buyer sues the employee for breach of contract, is the buyer likely to prevail?


A. Yes, because the buyer's promise to buy, bargained for and made in exchange for the employee's promise to sell, was good consideration even though it was expressly conditioned on an event that was not certain to occur.B. Yes, because any possible lack of consideration for the employee's promise to sell was expressly waived by the employee when the agreement was made.C. No, because mutuality of obligation between the parties was lacking when the agreement was made.D. No, because the condition of the parent company's approval of the contract was an essential part of the agreed exchange and was not in a signed writing.

Answer choice A is correct. In addition to an offer and acceptance, some legal detriment must be bargained for in order for a contract to exist. Consideration can take the form of a promise to do (or not do) something, a return promise to refrain from doing something legally permitted, or the actual performance of some act. The employee's first argument is incorrect because her conditioned promise to sell was valid consideration; thus, answer choice B and answer choice C are incorrect. Answer choice B is incorrect because consideration was not waived, either expressly or impliedly. Both parties actually provided consideration in the form of promises. Answer choice C is incorrect because there was mutuality of obligation, even though the buyer’s obligation was conditioned upon the parent company’s approval; in the event that the parent company did approve the contract, the buyer would have no discretion to choose not to perform. Answer choice D addresses the notion that the parent company's approval was an essential part of the agreement and was not in writing, which is only applicable if the buyer was backing out of its obligation to pay. Here, it is the employee, not the buyer, who is backing out, so the analysis of answer choice D is inapplicable to this fact pattern. The foregoing NCBE MBE question has been modified to reflect current NCBE stylistic approaches; the NCBE has not reviewed or endorsed this modification.


A famous chef entered into a written agreement with a friend, a well-known interior decorator respected for his revolutionary designs, in which the friend agreed, for a fixed fee, to design the interior of the famous chef's new restaurant, and, upon the famous chef's approval of the design plan, to decorate and furnish the restaurant accordingly. The agreement was silent as to assignment or delegation by either party. Before beginning the work, the friend sold his decorating business to a third party under an agreement in which the friend assigned to the third party, and the third party agreed to complete, the chef-friend contract. The third party, also an experienced decorator of excellent reputation, advised the chef of the assignment and supplied him with information confirming both the third party's financial responsibility and past commercial success.

If the famous chef allows the third party to perform and approves his design plan, but the third party fails without legal excuse to complete the decorating as agreed, against whom does the famous chef have an enforceable claim for breach of contract?


A. The friend only, because the friend's agreement with the third party did not discharge his duty to the famous chef, and the third party made no express promise to the famous chef.B. The third party only, because the friend's duty to the famous chef was discharged when the friend obtained a skilled decorator (the third party) to perform the chef-friend contract.C. The third party only, because the famous chef was an intended beneficiary of the friend-third party agreement, and the friend's duty to the famous chef was discharged when the famous chef permitted the third party to do the work and approved the third party's design.D. Either the friend or the third party, because 1) the friend's agreement with the third party did not discharge his duty to the famous chef, and 2) the famous chef was an intended beneficiary of the friend-third party agreement.

Answer choice D is correct. When obligations are delegated, the delegatee (here, the third party) is liable for nonperformance, but the delegator (here, the friend) is not automatically released from liability. Therefore, the famous chef can recover from either the friend or the third party for the third party’s nonperformance. Answer choice A is incorrect because, although it is true that the friend remains liable, the third party is not required to make an express promise to the famous chef to be liable. Answer choice B is incorrect because it does not address that, absent an express novation releasing the friend from liability, the famous chef may also recover from the friend. Answer choice C is incorrect because, although it is true that the famous chef was an intended beneficiary of the friend-third party agreement, it is incorrect that the friend’s duty to the famous chef was discharged.

A broker needed a certain rare coin to complete a set that he had contracted to assemble and sell to a collector. On February 1, the broker obtained such a coin from a third party in exchange for $1,000 and the broker's signed, written promise to re-deliver to the third party "not later than December 31 this year" a comparable specimen of the same kind of coin without charge to the third party. On February 2, the broker consummated sale of the complete set to the collector. On October 1, the market price of rare coins suddenly began a rapid, sustained rise; on October 15 the third party wrote the broker for assurance that the latter would timely meet his coin-replacement commitment. The broker replied, "In view of the surprising market, it seems unfair that I should have to replace your coin within the next few weeks." After receiving the broker's message on October 17, the third party telephoned the broker, who said, "I absolutely will not replace your coin until the market drops far below its present level." The third party then sued the broker on November 15 for the market value of a comparable replacement coin as promised by the broker in February. The trial began on December 1.

If the broker moves to dismiss the third party's complaint, which of the following is the third party's best argument in opposing the motion?


A. The third party's implied duty of good faith and fair dealing in enforcement of the contract required her to mitigate her losses on the rising market by suing promptly, as she did, after becoming reasonably apprehensive of a prospective breach by the broker.B. Although the doctrine of anticipatory breach is not applicable under the prevailing view if, at the time of repudiation, the repudiatee owes the repudiator no remaining duty of performance, the doctrine applies in this case because the third party, the repudiatee, remains potentially liable under an implied warranty that the coin advanced to the broker was genuine.C. When either party to a sale-of-goods contract repudiates with respect to a performance not yet due, the loss of which will substantially impair the value of the contract to the other, the aggrieved party may, in good faith, resort to any appropriate remedy for breach.D. Anticipatory repudiation, as a deliberate disruption without legal excuse of an ongoing contractual relationship between the parties, may be treated by the repudiatee at her election as a present tort, actionable at once.

Answer choice C is correct. Upon repudiation, the promisee can treat the repudiation as a breach and resort to any appropriate remedies. Answer choice A is incorrect, as good faith, fair dealing, and mitigation are all used out of context here. What matters is that the broker repudiated, and the third party is entitled to treat that as a breach and seek any appropriate remedies. Answer choice B is incorrect, as the U.C.C. permits treatment as a breach under these circumstances if the prospective loss will substantially impair the value of the contract for the aggrieved party. Answer choice D is incorrect, as this is a repudiation of a contract under contract law and not an actionable tort.


A father and his adult daughter encountered an old family friend on the street. The daughter said to the friend, "How about lending me $1,000 to buy a used car? I'll pay you back with interest one year from today." The father added, "And if she doesn't pay it back as promised, I will." The friend thereupon wrote out and handed to the daughter his personal check, payable to her, for $1,000, and the daughter subsequently used the funds to buy a used car. When the debt became due, both the daughter and the father refused to repay it, or any part of it. In an action by the friend against the father to recover $1,000 plus interest, which of the following statements would summarize the father's best defense?


A. He received no consideration for his conditional promise to the friend.B. His conditional promise to the friend was not to be performed in less than a year from the time it was made.C. His conditional promise to the friend was not made for the primary purpose of benefiting himself (the father).D. The loan by the friend was made without any agreement concerning the applicable interest rate.

Answer choice C is correct. This was a suretyship contract, which the Statute of Frauds generally requires to be in writing to be enforceable. There is an exception to this rule if the main purpose of the surety in agreeing to pay the debt of the principal is for the surety's own economic advantage, rather than for the principal's benefit. Here, the facts do not indicate any such motivation, and the father would be able to assert the Statute of Frauds.

A client consulted his lawyer about handling the sale of the client's building and asked the lawyer what her legal fee would be. The lawyer replied that her usual charge was $100 per hour and estimated that the legal work on behalf of the client would cost about $5,000 at that rate. The client said, "Okay; let's proceed with it," and the lawyer timely and successfully completed the work. Because of unexpected title problems, the lawyer reasonably spent 75 hours on the matter and shortly thereafter mailed the client a bill for $7,500 with a letter itemizing the work performed and time spent. The client responded by a letter expressing his good-faith belief that the lawyer had agreed to a total fee of no more than $5,000. The client enclosed a check in the amount of $5,000 payable to the lawyer and conspicuously marked, "Payment in full for legal services in connection with the sale of [the client's] building." Despite reading the "Payment in full..." language, the lawyer, without any notation of protest or reservation of rights, endorsed and deposited the check to her bank account. The check was duly paid by the client's bank. A few days later, the lawyer unsuccessfully demanded from the client payment of the $2,500 difference between the amount of her bill and the check and now sues the client for that difference.

What, if anything, can the lawyer recover from the client?


A. Nothing, because the risk of unexpected title problems in a real property transaction is properly allocable to the seller's attorney and thus to the lawyer in this case.B. Nothing, because the amount of the lawyer's fee was disputed in good faith by the client, and the lawyer impliedly agreed to an accord and satisfaction.C. $2,500, because the client agreed to an hourly rate for as many hours as the work reasonably required, and the sum of $5,000 was merely an estimate. D. The reasonable value of the lawyer's services in excess of $5,000, if any, because there was no specific agreement on the total amount of the lawyer's fee

Answer choice B is correct. This is an accord and satisfaction, and by endorsing and depositing the check, the lawyer impliedly agreed to it. Under an accord agreement, if a creditor agrees to accept a lesser amount in full satisfaction of the debt, the original debt is discharged only where there is some dispute either as to the validity of the debt or the amount of the debt, or where the payment is of a different type than called for under the original contract. Here, there was a clear dispute about the amount of the debt. By endorsing and depositing the check, the lawyer agreed to discharge the original debt and the accord contract. Answer choice A is incorrect, as the risk of title problems was not allocated to the lawyer. The contract called for $100 an hour, and that was owed. By endorsing and depositing the check, the lawyer agreed to an accord and satisfaction. Answer choice C is incorrect, as by endorsing and depositing the check, the lawyer agreed to discharge the original debt. Answer choice D is incorrect, as there was a specific agreement with regard to the total amount of the lawyer's fee ($100 an hour). The lawyer

An exterminator, who has been in the exterminating business for fifteen years and has a fine reputation, contracts to provide services to a clothing shop with a moth infestation. The contract has no provision regarding assignment.

If the exterminator assigns the contract to the contractor and thereafter the contractor does not meet the contract specifications in providing services to the clothing shop, the shop owner


A. has a cause of action against the exterminator for damages.B. has a cause of action only against the contractor for damages.C. has a cause of action against the exterminator for damages only after he has first exhausted his remedies against the contractor.D. does not have a cause of action against the exterminator for damages, because he waived his rights against the exterminator by permitting the contractor to perform the work.

Answer choice A is correct. In either an assignment or a delegation, the assignor or delegator is not released from liability and recovery can be had against him if performance is not made. Here, the exterminator is not released from liability. Answer choice B is incorrect because the shop owner can sue both the exterminator and the contractor for damages, and there is no indication that there was a novation of the contract releasing the exterminator from liability. Answer choice C is incorrect because it misstates the exterminator's primary responsibility in assignment or delegation. Answer choice D is incorrect because there is no waiver here. Parties to a contract generally retain the power to assign and delegate rights and duties to third parties unless doing so materially increases the duty or risk of the obligor or materially reduces the obligor’s chance of obtaining performance. There is no such indication in the facts. The shop owner would be obliged to accept performance by the contractor and he may sue either the exterminator or the contractor for damages arising from the contractor's faulty performance.

On July 15, in a writing signed by a manufacturer and a customer, a retail furniture store, the manufacturer agreed to deliver to the customer on August 15 five storage cabinets from inventory, for a total price of $5,000 to be paid on delivery. On August 1, the two parties orally agreed to postpone the delivery date to August 20. On August 20, the manufacturer tendered the cabinets to the customer, who refused to accept or pay for them on the ground that they were not tendered on August 15, even though they otherwise met the contract specifications.

Assuming that all appropriate defenses are seasonably raised, will the manufacturer succeed in an action against the customer for breach of contract?


A. Yes, because neither the July 15 agreement nor the August 1 agreement was required to be in writing.B. Yes, because the August 1 agreement operated as a waiver of the August 15 delivery term.C. No, because there was no consideration to support the August 1 agreement.D. No, because the parol evidence rule will prevent proof of the August 1 agreement.


Answer choice B is correct. The parties are both merchants with a contract for the sale of goods, which falls within the U.C.C. Although modification to a contract not within the U.C.C. requires additional consideration, no consideration is required to modify a contract for the sale of goods, as long as it was made in good faith. The creation of a contract for goods exceeding $500 is subject to the Statute of Frauds writing requirement, but modifications of a properly executed agreement are not required to be in writing, as long as the modification does not affect the subject matter or quantity. In this case, the modification was to the date of delivery, not the subject matter or quantity to be delivered, so the modification is valid. Note that answer D tests your understanding of the parol evidence rule, which applies to prior or contemporaneous agreements, not subsequent modifications.